2019 AMC 12B Problems/Problem 22

Revision as of 17:38, 14 February 2019 by Expilncalc (talk | contribs) (Problem: Added problem statement.)

Problem

Define a sequence recursively by $x_0 = 5$ and

$x_{n+1} = \frac{x_n^2 + 5x_n + 4}{x_n + 6}$

for all nonnegative integers $n$. Let $m$ be the least positive integer such that $x_m \leq 4 + \frac{1}{2^{20}}$.

In which of the following intervals does $m$ lie?

Solution

See Also

2019 AMC 12B (ProblemsAnswer KeyResources)
Preceded by
Problem 21
Followed by
Problem 23
1 2 3 4 5 6 7 8 9 10 11 12 13 14 15 16 17 18 19 20 21 22 23 24 25
All AMC 12 Problems and Solutions